Multiply. Simplify your answer and write as a mixed or whole number.

6 1/3 ⋅ 2

Answers

Answer 1
38/3 or 12 2/3
Pls mark brainliest

Related Questions

What is X?
I’m at a loss

What is X?Im at a loss

Answers

Answer:

9

Step-by-step explanation:

5x-3= 2x+24

5x=2x+27

3x=27

X=9

Answer: A.  9

Step-by-step explanation:

The diagonal is bisected by the other diagonal.  So the 2 parts of the diagonal are equal

5x - 3 =  2x + 24                 >Bring like terms to 1 side

3x = 27                                >Divide both sides by 3

x = 9

if m<xyz = 58 and m<wxz = 51 find m<wzx​

Answers

Answer:

m<wzx = 71

Step-by-step explanation:

Assuming these are interior angles of a triangle.

The sum of all three interior angles of a triangle is always 180 degrees, therefore:

m<xyz + m<wxz + m<wzx = 180

Substitute our values:

58 + 51 + m<wzx = 180

m<wzx = 180 - 58 - 51

m<wzx = 71

A sponsor created this box and whisker plot to show the ages of the 120 people seated in the first five rows at the concert. What was the median age of the 120 people sitting in the first five rows

A sponsor created this box and whisker plot to show the ages of the 120 people seated in the first five

Answers

The median age of the 120 people seated in the first five rows of the concert was 31.

What is value?

Value is an important concept in economics and business. It is the worth of goods, services, or assets that is determined by the market or by an individual. It can be measured in terms of money, time, or effort. Value is also used to describe a person’s sense of worth or importance. Values can be seen as the principles, standards, or goals that guide an individual’s actions and decisions. Ultimately, value is subjective and is held by each person and can change over time.

The box and whisker plot provided by the sponsor shows the ages of the 120 people seated in the first five rows of the concert. The box and whisker plot shows a minimum value of 18, a maximum value of 45, a median of 31, a first quartile of 24, and a third quartile of 37. This indicates that 50% of the 120 people in the first five rows had an age of 31 or below, and 50% of the 120 people had an age of 37 or above. The median age of the 120 people seated in the first five rows of the concert was 31.

To know more about value click-
http://brainly.com/question/843074
#SPJ1

What is the value of n?

What is the value of n?

Answers

Answer:

A.  95°

Step-by-step explanation:

see attached

What is the value of n?

Two pieces of equipment were purchased for a total of $2000. If one piece cost $810 more than the other, find the price of the less expensive piece of equipment.

Answers

Answer:

595.

Step-by-step explanation:

Take $2000 and subtract $810 from it. You now have the total of the two pieces of equipment without the extra pricing. You should get $1,190. Then divide that number by 2. You will get $595.

1st Piece of Equipment = $595

2nd Piece of Equipment = $1,405

2nd Piece is $810 more expensive and the two pieces combined is $2,000.

Use a geometric tool to draw a circle. Draw and measure a radius and a diameter of the circle .

Answers

Answer:

Attached is an example of a circle with a radius of 5 and a diameter of 10.

If this answer helped you, please leave a thanks or a Brainliest!!!

Have a GREAT day!!!

Use a geometric tool to draw a circle. Draw and measure a radius and a diameter of the circle .

Se dispone de 800 000 para invertir. Una cuenta paga el 18% de interés anual, y otra paga el 21%. ¿Cuánto debe invertirse en cada cuenta para ganar 150 000 en intereses en un año?

Answers

Se debe depositar $ 600 000 en la cuenta con 18 % de interés anual y $ 200 000 en la cuenta con 21 % de interés anual para recibir $ 150 000 en intereses.

¿Cuánto se debe invertir en cada cuenta para alcanzar las ganancias deseadas en un período dado?

En este problema tenemos un depósito repartido en dos cuentas, que adquiere ganancias de manera continua en el tiempo. En consecuencia, tenemos por interés compuesto la siguiente ecuación a resolver:

x · (18/100) + (800 000 - x) · (21/100) = 150 000

168 000 - (3/100) · x = 150 000

(3/100) · x = 18 000

x = 600 000

Se debe depositar $ 600 000 en la cuenta con 18 % de interés anual y $ 200 000 en la cuenta con 21 % de interés anual para recibir $ 150 000 en intereses.

Para aprender más sobre el interés compuesto: https://brainly.com/question/23137156

#SPJ1

find a quadratic equation in X whose roots are -2 and 3/4​

Answers

Answer:

4x² + 5x - 6  = 0

Step-by-step explanation:

Quadratic equation:

             \(\boxed{x^2-(sum \ of \ roots)x+product \ of \ roots=0}\)

    \(Sum \ of \ roots = -2 + \dfrac{3}{4}\)

                           \(= \dfrac{-2*4}{1*4}+\dfrac{3}{4}\\\\=\dfrac{-8}{4}+\dfrac{3}{4}\\\\=\dfrac{-5}{4}\\\\\)

\(\sf product \ of \ roots =-2 *\dfrac{3}{4}\)

                         \(=\dfrac{-6}{4}\)

Quadratic equation:

      \(x^2 - \left(\dfrac{-5}{4}\right)x+\left(\dfrac{-6}{4}\right)=0\)

Multiply the entire equation by 4,

        4x² - (-5)x + (-6) = 0

         4x² + 5x - 6  = 0

The quadratic formula states that the roots of the equation ax^2 + bx + c = 0 are given by (-b ± √(b^2 - 4ac))/(2a). To find a quadratic equation whose roots are -2 and 3/4, we can set these values equal to (-b ± √(b^2 - 4ac))/(2a) and solve for the values of a, b, and c.

One such equation is (x+2)(x-3/4) = 0. Expanding this product, we get x^2 - x/2 + 3/8 = 0

Another equation is x^2 - 4x + 8 = 0 (by Vieta's Formulas)

Simplify each side of 4a+ 5a-2=5+3-1

Answers

Answer:

9a-2=7

Step-by-step explanation:

4a+ 5a-2=5+3-1

To simplify each side, combine like terms.

4a+5a = 9a

5+3-1 = 7

The equation becomes:

9a-2=7

what would be the slope?​

what would be the slope?

Answers

I think 8, because 1/8

Check whether x = -3 is a solution to the equation. Justify your result.
x/3 - 6 - 5 = -12

(show work)

Answers

The value of x i.e., x = -3 is a solution to the equation which is (x/3) - 6 - 5 = -12.

What do you mean by algebraic expressions ?

Algebraic expression is an equation which consists of variables and the arithmetic operations such as division , multiplication , etc.

It is given that x = -3 is a solution to the equation which is x/3 - 6 - 5 = -12 .

This equation is a form of algebraic expression which is expression which consist of a variable and arithmetic operations.

Let's check out that whether this equation is true for x = -3 . To be true both sides of the equation must be equal.

So , the equation is :

(x/-3) - 6 - 5 = -12

If we put x = -3 in the above equation :

(3/-3) - 6 - 5 = -12

-1 - 6 - 5 = -12

-1 - 11 = - 12

-12 = -12

So , for x = -3 equation is satisfied.

Therefore , the value of x i.e., x = -3 is a solution to the equation which is (x/3) - 6 - 5 = -12.

Learn more about algebraic expressions here :

brainly.com/question/2164351

#SPJ1

Please solve this question!! Please help me

Please solve this question!! Please help me

Answers

\(\frac{x+3}{x-3}-\frac{x-2}{x+2}\)

In order to simplify the expression, start by solving the subtraction of fractions in the form

\(\frac{a}{b}-\frac{c}{d}=\frac{ad-bc}{bd}\)

then,

\(undefined\)

Dylan has a pitcher with 1.65 L of orange juice. He pours out 0.2 L
of the juice. Then he adds some sparkling water to the pitcher to
make orangeade. He ends up with 1.9 L of orangeade. Solve the
equation 1.65 -0.2 + x = 1.9 to find the amount of sparkling
water, x, Dylan adds to the pitcher. Show your work.

Answers

Answer:

Step-by-step explanation:

fork knife

The FIRST ANSWER GETS BrAINLIEST

The FIRST ANSWER GETS BrAINLIEST

Answers

Answer:

me

Step-by-step explanation:

Printer A prints 100 pages for $23.99. Printer B prints 275 sheets for $63.99. Which printer has the better rate of cost per page?

Printer A, because the approximate rate of Printer A, $0.24 per page, is greater than the approximate rate of Printer B, $0.23 per page
Printer A, because the approximate rate of Printer A, $4.16 per page, is less than the approximate rate of Printer B, $4.30 per page
Printer B, because the approximate rate of Printer A, $0.24 per page, is greater than the approximate rate of Printer B, $0.23 per page
Printer B, because the approximate rate of Printer A, $4.16 per page, is less than the approximate rate of Printer B, $4.30 per page
PLEASE HURRY I NEEED THIS RN :)

Answers

Printer A has the better rate of cost per page because the approximate rate of Printer A, $0.24 per page, is greater than the approximate rate of Printer B, $0.23 per page.

How to calculate the rate?

Rate demonstrates how many times one number can fit into another number. It contrast two numbers by ordinarily dividing them. A/B will be the formula if one is comparing one data point (A) to another data point (B).

In this case, Printer A prints 100 pages for $23.99. The rate is:

\(\sf = \dfrac{\$23.99}{100}\)

\(\sf = 0.2399\thickapprox\$0.24 \ per \ page\)

Printer B prints 275 sheets for $63.99. The rate is:

\(\sf = \dfrac{\$63.99}{275}\)

\(\sf = 0.2327\thickapprox\$0.23 \ per \ page\)

Therefore, based on the above calculations, we can see that Printer A has the better rate of cost per page because the approximate rate of Printer A, $0.24 per page, is greater than the approximate rate of Printer B, $0.23 per page.

So option (A) is correct.

Learn more about rate on:

brainly.com/question/25537936

Choose the table that represents g(x) = 3⋅f(x) when f(x) = x − 1

Answers

If f(x) = x-1 then your answer would be -2 -3 -4

A bag contains 11 apples and 6 oranges. If you select 6 pieces of fruit without looking, how many ways can you select exactly 5 apples

Answers

Answer:

you have a 11/17 percent chance of getting an apple

Step-by-step explanation:

if there are 11 apples and 6 oranges. 11 out of 17 times you will get an apple

Need a step by step answer please

Need a step by step answer please

Answers

Answer:

x=10

Step-by-step explanation:

x+2+x=22

combine like terms x + x to get 2x:

2x + 2 = 22

subtract 2 on both sides:

2x = 20

divide by 2 on both sides to isolate x:

x=10

with a 5% sale discount the price of an item is $38 what was the original price​

Answers

Answer:

\(38 \div5 \\ \)

the original price is $40

d. 2/5 (j + 40) = -4

Answers

Answer:

-50

i have made it in the above picture

hope it helps

d. 2/5 (j + 40) = -4

Step-by-step explanation:

2/5 (j + 40) = -4

(j + 40) =-10

j=-50

Hope it helps

PLEASE HELP ME 1. A different pool had an area that is of the form
▢ × 102 + ▢ × 101 + 6
and that can be written in the form x3 ,
where x is a whole number.

A) Decide what your number could be.



B) What is the perfect square number that is closest to the number you chose? What would the side length of a square pool with that area be?


C) Estimate the side length of a square pool with the area you chose in part a).

Answers

The solutions to the questions are

The number we are using is 5The perfect square number closest to the area is 1024The estimated side length is 32

What are areas?

The area of a shape is the amount of space on the shape

For most regular quadrilaterals, you multiply the side lengths to determine the area

The number to use

The area expression is given as

Area = ▢ × 102 + ▢ × 101 + 6

From the question, we can use any number

However, this number must be positive

Assume the number is 5

So, we have

Area = 5 × 102 + 5 × 101 + 6

The perfect square number closest to the result

In (a), we have:

Area = 5 × 102 + 5 × 101 + 6

Evaluate

Area = 1021

The perfect square number closest to this is

Closest = 1024

The estimated side length

In (b), we have

Closest = 1024

Rewrite as

Area = 1024

Take the square roots

Length = 32

Read more about areas at

https://brainly.com/question/16184187

#SPJ1

Which of the following is a factor of 9x^2+21+10

Which of the following is a factor of 9x^2+21+10

Answers

Answer: 3x+5

Step-by-step explanation:

trust me  i had that question for home work

Please answer ASAP. Do not mind the purple thing.

Please answer ASAP. Do not mind the purple thing.

Answers

Answer:

1 4

2 8

3 12

6 24

for next year, its 32

Step-by-step explanation:

cauz 8 divided by 2 is 4

Also since it goes up by four, 3 is 12 and 6 is 24

15) Fred drank 1/3 of a cup of milk at breakfast and 2/3 of a cup of milk at dinner. In total, how many cups
of milk did Fred drink today?

Answers

Answer: Fred had 1 cup of milk to drink.

Step-by-step explanation: 1/3 + 2/3 equals 3/3 or 1

Does anyone know this?

Does anyone know this?

Answers

Answer:

i think c or b I D K

Step-by-step explanation:

Find the area and the perimeter of a rectangle with side lengths of 4 1/5in. and 4 2/7 in.

Answers

the area is simply the product of its width and length, so hmmm before doing so, let's change all the mixed fractions to improper fractions and then multiply.

\(\stackrel{mixed}{4\frac{1}{5}}\implies \cfrac{4\cdot 5+1}{5}\implies \stackrel{improper}{\cfrac{21}{5}} ~\hfill \stackrel{mixed}{4\frac{2}{7}} \implies \cfrac{4\cdot 7+2}{7} \implies \stackrel{improper}{\cfrac{30}{7}} \\\\[-0.35em] ~\dotfill\\\\ \cfrac{21}{5}\cdot \cfrac{30}{7}\implies \cfrac{21}{7}\cdot \cfrac{30}{5}\implies \cfrac{3}{1}\cdot \cfrac{6}{1}\implies \text{\LARGE 18}~in^2\)

The area of the rectangle is 18 \(inch^{2}\) and the perimeter is  \(16\frac{34}{35}\) inches.

We know that, Area of a rectangle = l * b

        and Perimeter of a rectangle = 2(l + b)

 Where, l = length of the rectangle

             b = breadth of the rectangle

According to the question, l = \(4\frac{1}{5}\) inch = \(\frac{21}{5}\) inch

                                             b = \(4\frac{2}{7}\) inch = \(\frac{30}{7}\) inch    

Area of the given rectangle = l* b

                                              =   \(\frac{21}{5}\) * \(\frac{30}{7}\)

                                              = 18 \(inch^{2}\)

The perimeter of the given rectangle = 2(l + b)

                                                              = 2( \(\frac{21}{5}\) + \(\frac{30}{7}\))

                                                              = 2 (\(\frac{297}{35}\))

                                                              = \(\frac{594}{35}\)

                                                              = \(16\frac{34}{35}\) inch

Hence, the area of the rectangle = 18 \(inch^{2}\)

and the perimeter =  \(16\frac{34}{35}\) inch

Learn more about the Area and the Perimeter of a rectangle:

brainly.com/question/11957651

What's the first step of purchasing insurance?
O contacting a company or an agent
O signing a contract with a company
O shopping around for the best deal
O receiving proof of an insurance policy

Answers

Answer:

C - Shopping around for the best deal

Step-by-step explanation:

You might be surprised by certain insurance deals, but by shopping around, you can find some insurance deals which are good and affordable.

‼️‼️‼️‼️‼️‼️‼️‼️HELP PLEASE ASAP ‼️‼️‼️‼️‼️‼️

HELP PLEASE ASAP

Answers

Answer:

C I think

Step-by-step explanation:

Answer:

A, f(x) ≥ -36

Step-by-step explanation:

The range is all the possible y values, also called f(x) values.

In this case, the range of the graph extends from -36 to infinity, so the correct answer is f(x) ≥ -36.

Let me know if this helps!

Find a coordinate on the line.

y - 5 = 8(x + 2)
Question 3 options:


(8, 2)


(2, -5)


(-2, 5)


(-5, 2)

Answers

Answer:

(-2,5)

Step-by-step explanation:

Please Answer with Explanation

Please Answer with Explanation

Answers

Answer:

a.

Step-by-step explanation:

it is not clear what your teacher discussed and wants from you here.

formally, all triangles are similar, as the description says. and the sequence of the vertexes in the "name" does not make any difference in identifying the triangle. this would be only important when identifying a specific angle (the center letter). in other words : a triangle is identified by naming its three corners in any sequence. simply because for the similarity concern you can twist and turn the triangle until it has the same orientation as the other triangle to compare.

so, again, a very strange question from your teacher. many greetings from me.

as explained, formally all 4 statements are correct.

but in order to identify one, it has to be a.

because the sequence of the corners are the same.

in QTS the side QT (the shortest of the 3 sides) is named first. and TS (the longest of the 3 sides) is named last.

in PQT we have the same sequence. PQ is the shortest side first, and QT is the longest side last.

no other answer option has the same sequence on the left and the right side.

but again, this is actually irrelevant for the similarity relation.

e.g. triangle RQS = triangle SQR, they are just twisted and turned images of the same triangle.

Other Questions
Im confused about the whole thing, can anyone help me out? brady- (brady/cardia; brady/kinesia) means: when pressure of a gas increases then the volume must calculate the percent dissociation of -chlorobutanoic acid in a aqueous solution of the stuff. you may find some useful data in the aleks data resource. round your answer to significant digits. write an equation for a rational function with vertical asymptote at x=-1 and x=2x inter at x=-3 and x=-6 horizontal asymptote at y=3 Why the idea of limited resources is such a major concern in today's economy? king menes ruled egypt from his capital at hyksos. truefalse Help me please I dont know what to do cindy has recurring attacks where she has a pounding heart, feels she can't breathe enough air in, and thinks she might die. which of the following does she likely have? group of answer choices Franz Schubert was affectionatelycalled "little mushroom" by hisfrierids due toA. his tall, strong build.B. his enormous head.C. his short, stocky build.Need answer ASAP 2. Describe Chihiro at the beginning of the film. Are her feelings justified? Why orwhy not? Biocatalysis is:O A form of genetic sequencingO A form of genetic modificationO Speeding up or aiding a chemical reaction using a biological enzymeA means of mutating the genome of a simple organism Question 27Goodie Corporation produces goods that are very mature in their product life cycles. Goodie Corporation is expected to have per share FCFE in year 1 of $2.00, per share FCFE of $1.50 in year 2, and per share FCFE of $1.00 in year 3. After year 3, per share FCFE is expected to decline at a rate of 1% per year. An appropriate required rate of return for the stock is 10%. The stock should be worth __________ today.A.$9.00B.$101.57C.$10.57D.$22.22 Which client should the nurse anticipate will be at greatest risk for alteration in quality of life as a result of loss? You are at a professional football game. You look across from your seat and notice the other side of the stadium and all of the people in their seats. You close your eyes, and for a brief second, you see an accurate afterimage of that view in your mind. This is an example of ______. echoic memory working memory sensory memory short-term memory Valence electrons are electrons in the ? of an atom. Ryan has a bank account balance of --$17.62. As soon as he realizes this, he deposits $26.50 in the account. What is Ryan's bank account balance after the deposit is made? Enter your answer in the box. 69 Who is responsible for verifying the accuracy of information presented in a Franchise Disclosure Document (FDD)?A) A Federal governmental agencyB) The person or company reading the FDDC) The person or company preparing the FDDD) A governmental agency operating in the state in which the FDD was filed help me please....... The biggest challenge for researching a speech is picking a topic that is